[escepticos] Pregunta sobre microondas

Antonio Larrosa antonio.larrosa en gmail.com
Sab Ene 5 18:41:17 WET 2008


On Friday 04 January 2008 08:20:29 Eloy Anguiano Rey wrote:
> El vie, 04-01-2008 a las 03:27 +0100, Antonio Larrosa escribió:
> > Buenas,
> >
> > Aunque no participe mucho en la lista (intento seguirla pero el tiempo no
> > siempre está de mi parte), me voy a permitir el lujo de preguntar una
> > duda a los expertos de por aquí.
>
> Hombre Antonio, qué es de tu vida. ;-)

Pues bien liado como siempre, pero sacando huecos para que no todo sea 
trabajar.

> Para los linuxeros aquí tenéis a un "developer, developer,
> developer" (emulando a Balmer).

jeje (aunque a Balmer mejor emularlo lo menos posible :) )

> Ya te ha contestado muy bien Català pero he de puntualizar algunas
> cosas. Como es una rendija y un agujero realmente sí se escapan las
> microondas. Ahora, eso sí, la potencia que se escapa es proporcional al
> sen²(f) donde f es la mitad del ángulo que forma el centro de la rendija
> con los puntos que se encuentran en los bordes a unos 6 cm (la mitad de
> la longitud de onda) de este punto a cada lado de la rendija. Como el
> ángulo es pequeño puedes tomar como ángulo el que se forma con las dos
> equinas superiores (o inferiores). Además, como ese ángulo es muy
> pequeño, la potencia que se escapa es muy pequeña. Y también te lo ha
> comentado muy claro, no son ionizantes y, por tanto, sólo calientan. Si
> no te pones cerca, no hay problema. Además, a partir de la rendija, la
> potencia disminuye más o menos con el cuadrado de la distancia. No es
> exactamente con el cuadrado porque la emisión no es puntual.
>
> Por lo demás, ninguna radiación electromagnética puede "atravesar
> agujeros" en un material opaco que sean inferiores a la mitad de su
> longitud de onda.

Ahá, esa parece una regla interesante, gracias.

Un saludo (extiéndelo a toda la gente del CRL, Eloy)

-- 
Antonio Larrosa


Más información sobre la lista de distribución Escepticos